Stay ahead of learning milestones! Enroll in a class over the summer!

G
Topic
First Poster
Last Poster
USAMO 1985 #2
Mrdavid445   6
N 19 minutes ago by anticodon
Determine each real root of \[x^4-(2\cdot10^{10}+1)x^2-x+10^{20}+10^{10}-1=0\]correct to four decimal places.
6 replies
Mrdavid445
Jul 26, 2011
anticodon
19 minutes ago
Inequality with rational function
MathMystic33   3
N an hour ago by ariopro1387
Source: Macedonian Mathematical Olympiad 2025 Problem 2
Let \( n > 2 \) be an integer, \( k > 1 \) a real number, and \( x_1, x_2, \ldots, x_n \) be positive real numbers such that \( x_1 \cdot x_2 \cdots x_n = 1 \). Prove that:

\[
\frac{1 + x_1^k}{1 + x_2} + \frac{1 + x_2^k}{1 + x_3} + \cdots + \frac{1 + x_n^k}{1 + x_1} \geq n.
\]
When does equality hold?
3 replies
MathMystic33
4 hours ago
ariopro1387
an hour ago
No more topics!
Find the value
sqing   9
N Apr 20, 2025 by sqing
Source: 2025 Tsinghua University
Let $A= \lim_{n\to\infty}\tan^n\left(\frac{\pi}{4}+\frac{1}{n}\right)  . $ Find the value of $[100A] .$
9 replies
sqing
Apr 20, 2025
sqing
Apr 20, 2025
Find the value
G H J
G H BBookmark kLocked kLocked NReply
Source: 2025 Tsinghua University
The post below has been deleted. Click to close.
This post has been deleted. Click here to see post.
sqing
42154 posts
#1
Y by
Let $A= \lim_{n\to\infty}\tan^n\left(\frac{\pi}{4}+\frac{1}{n}\right)  . $ Find the value of $[100A] .$
This post has been edited 2 times. Last edited by sqing, Apr 20, 2025, 1:20 PM
Z K Y
The post below has been deleted. Click to close.
This post has been deleted. Click here to see post.
pco
23511 posts
#3
Y by
sqing wrote:
Let $A=\lim_{n\to\infty}(\frac{\pi}{4}+\frac{1}{n})^n  . $ Find the value of $[100A] .$
$A=0$ and so $\boxed{\lfloor 100A\rfloor=0}$
Z K Y
The post below has been deleted. Click to close.
This post has been deleted. Click here to see post.
Quantum-Phantom
273 posts
#4
Y by
It should be
\[A=\lim_{n\to\infty}\tan^n\left(\frac{\pi}{4}+\frac{1}{n}\right).\]
Z K Y
The post below has been deleted. Click to close.
This post has been deleted. Click here to see post.
sqing
42154 posts
#5
Y by
Thank you very much.
Z K Y
The post below has been deleted. Click to close.
This post has been deleted. Click here to see post.
sqing
42154 posts
#6
Y by
Sorry.Thank pco.
Z K Y
The post below has been deleted. Click to close.
This post has been deleted. Click here to see post.
SunnyEvan
119 posts
#7
Y by
Quantum-Phantom wrote:
It should be
\[A=\lim_{n\to\infty}\tan^n\left(\frac{\pi}{4}+\frac{1}{n}\right).\]

$$ A=\lim_{n\to\infty}\tan^n\left(\frac{\pi}{4}+\frac{1}{n}\right) =e^{\lim_{n\to\infty}n ln\tan(\frac{\pi}{4}+\frac{1}{n})} $$let $t=\frac{1}{n}$
$$ e^{\lim_{n\to\infty}n ln\tan(\frac{\pi}{4}+\frac{1}{n})} = e^{\lim_{t\to 0} \frac{ln\tan(\frac{\pi}{4}+t)}{t}} = e^{\lim_{t\to 0} \frac{1}{tan(\frac{\pi}{4}+t)}(sec(\frac{\pi}{4}+t))^2}= e^{\lim_{t\to 0} \frac{1}{sin(\frac{\pi}{4}+t)cos(\frac{\pi}{4}+t)}}=e^2$$$[100A] =738$
This post has been edited 3 times. Last edited by SunnyEvan, Apr 20, 2025, 1:53 PM
Z K Y
The post below has been deleted. Click to close.
This post has been deleted. Click here to see post.
sqing
42154 posts
#9
Y by
Prove that $$\lim_{n\to\infty}\tan^n\left(\frac{\pi}{4}+\frac{1}{n}\right) = e^2$$
Z K Y
The post below has been deleted. Click to close.
This post has been deleted. Click here to see post.
sqing
42154 posts
#10
Y by
sqing wrote:
Prove that $$\lim_{n\to\infty}\tan^n\left(\frac{\pi}{4}+\frac{1}{n}\right) = e^2$$
https://artofproblemsolving.com/community/c7h1643781p10364775
https://math.stackexchange.com/questions/1750591/limit-of-tan-function?noredirect=1
*
Z K Y
The post below has been deleted. Click to close.
This post has been deleted. Click here to see post.
SunnyEvan
119 posts
#11
Y by
sqing wrote:

Thank you very much !@Sqing . :-D Hospital always help with lots of problems .
This post has been edited 1 time. Last edited by SunnyEvan, Apr 20, 2025, 2:02 PM
Z K Y
The post below has been deleted. Click to close.
This post has been deleted. Click here to see post.
sqing
42154 posts
#12
Y by
Thanks.
Attachments:
Z K Y
N Quick Reply
G
H
=
a